Page 1 of 1

Don't Intersect Please :D

Posted: Tue Feb 28, 2017 7:19 pm
by Thamim Zahin
Given $2n$ points in the plane with no three collinear, show that it is possible to pair them up in such a way that the $n$ line segments joining paired points do not intersect.

Re: Don't Intersect Please :D

Posted: Sun Mar 05, 2017 4:43 pm
by ahmedittihad
Take the convex hull and select two consecutive points and pair them up. Then take the next convex hull and do the algorithm. We eventually will get what we desire.

Re: Don't Intersect Please :D

Posted: Sun Mar 05, 2017 6:42 pm
by Thamim Zahin
Simple polygon will also do the trick. Isn't? If we put a super big rubber-band, it will result us a simple polygon.

Re: Don't Intersect Please :D

Posted: Sun Mar 05, 2017 10:41 pm
by Absur Khan Siam
Actually,@thamimzahin,the idea which you have is the main concept of convex hull.
http://mathworld.wolfram.com/ConvexHull.html

Re: Don't Intersect Please :D

Posted: Sun Mar 05, 2017 11:41 pm
by ahmedittihad
@thamimzahin I seem to remember to have taught you the concept of a convex hull. Did you forget already? :lol:

Re: Don't Intersect Please :D

Posted: Mon Mar 06, 2017 12:03 pm
by Absur Khan Siam
I think 70% or more are introduced to convex hull by this 'rubber-band' concept.:D

Re: Don't Intersect Please :D

Posted: Mon Mar 06, 2017 4:11 pm
by ahmedittihad
Related Problem,
Given $2n$ points in a plane with no three collinear, with $n$ red points and $n$ blue points, prove that
there exists a pairing of the red and blue points such that the $n$ segments joining each pair are pairwise
non-intersecting.

Re: Don't Intersect Please :D

Posted: Mon Mar 06, 2017 6:13 pm
by Thanic Nur Samin
Thamim Zahin wrote:Given $2n$ points in the plane with no three collinear, show that it is possible to pair them up in such a way that the $n$ line segments joining paired points do not intersect.
Here is my take.

Take all possible pairings of the points, and select the one with minimum sum of the length of the segments. Due to the triangle inequality, the segments do not intersect.

The argument works in Ittihad's problem as well.